Jump to content

Kamii0909's Content

There have been 155 items by Kamii0909 (Search limited from 04-06-2020)



Sort by                Order  

#662270 Chứng minh $G,G_1,G_2$ thẳng hàng.

Posted by Kamii0909 on 17-11-2016 - 22:32 in Hình học

Ta có $3\overrightarrow{GG_{1}}+3\overrightarrow{GG_{2}}=\sum\overrightarrow{GA_{1}}+\sum\overrightarrow{GA_{2}}=\overrightarrow0$

nên $G,G_{1},G{2}$ thẳng hàng. Cách sử dụng vecto. Bạn tham khảo




#658547 Cho tam giác ABC vuông tại A.Trung tuyến AM,BN vuông góc với nhau.Tính tỉ số...

Posted by Kamii0909 on 20-10-2016 - 18:06 in Hình học

goi giao diem la G

GD=x thì AG=2x suy ra BM=NB=NC=3x nên BC=6x

suy ra BG2=8x

AB2=AG2+GNsuy ra AB2=12x2

AN2=5x

từ đó tìm tỷ số lượng giác của B theo x

D ở đâu vậy bạn 




#662553 Chứng minh $I,E,F$ thẳng hàng .

Posted by Kamii0909 on 20-11-2016 - 20:54 in Hình học phẳng

Đề có đúng ko bạn. Mình vẽ nó sai



#660734 CMR: $S_{n+3}=S_{n+2}+S_{n+1}+S_{n...

Posted by Kamii0909 on 05-11-2016 - 22:56 in Đại số

Đa thức này có nghiệm duy nhất mà



#661703 Phương trình nghiệm nguyên

Posted by Kamii0909 on 12-11-2016 - 23:51 in Số học

Mình lý luận không được chắc chắn lắm có gì sai sót mong bạn chỉ giúp.

đặt $a=x+y,b=xy$

thì $a^2|a^3-3ab-b^2$ hay $a^2|3ab+b^2$

Từ đó $ka^2-3ab-b^2=0$

Coi đây là phương trình bậc 2 ẩn $a$ thì 

$\Delta =b^2(4k+9)$ nên $4k+9$ phải là số chính phương lẻ.

Từ đó tính được $k=m^2+m-2$

thay vào công thức nghiệm được $a= \frac{b}{m-1}$ hoặc $a= \frac{b}{m+2}$

Cả 2 đều chứng tỏ $a|b$ hay $x+y|xy$ nghĩa là $xy=p(x+y)$

Từ đề bài thì tử số phải không âm hay

$x+y \geq 3p+p^2$

Nếu $x=y$ thì $p= \frac{x}{2}$ thay vào có $x \leq 2$

$x=y=1$ KTM, $x=y=2$ TM.

TH còn lại xử lí như trên.




#675570 $$\prod \left( \dfrac{a}{b}+2...

Posted by Kamii0909 on 28-03-2017 - 22:52 in Bất đẳng thức - Cực trị

Cho các số thực dương $a,b,c$. Cmr
$$ ( \dfrac{a}{b}+2)( \dfrac{b}{c} +2)( \dfrac{c}{a}+2 ) + \dfrac{117(ab+bc+ca)}{4(a^2+b^2+c^2)} \geq \frac{107}{2}$$



#661638 Chứng minh rằng $\sum \frac{x^{4}+y^{4...

Posted by Kamii0909 on 12-11-2016 - 14:16 in Số học

Bài này nên đưa vào mục bất đẳng thức chứ nhỉ.
Có thể giải bằng Holder như sau(không hay lắm)
$(x^4 +y^4)^3(1+1) \geq (x^3+y^3)^4$
và $(x^3+y^3)(1+1)(1+1) \geq (x+y)^3$
Từ đó ta có
$\frac{x^4+y^4}{x^3+y^3} \geq \frac{x+y}{2}$
Cộng lại ta cũng có đpcm



#667463 $BM,CN,PD$ đồng quy

Posted by Kamii0909 on 07-01-2017 - 15:24 in Hình học

Cho tam giác $ABC$ có đường tròn nội tiếp $(I)$ tiếp xúc $BC,CA,AB$ tại $D,E,F$. Đường tròn $(PBC)$ tiếp xúc $(I)$ tại $P$. Gọi $M,N$ là trung điểm $DE,DF$. Chứng minh rằng $PD,BM,CN$ đồng quy.



#658555 Tâm đường tròn ngoại tiếp tam giác ACD chạy trên đường nào?

Posted by Kamii0909 on 20-10-2016 - 18:45 in Hình học

Lấy Q đối xứng với B qua H 
Do AHMD,HCMB là các tứ giác nội tiếp và theo tính chất đối xứng nên 

$\widehat{ADC}=\widehat{ADH}=\widehat{AMH}=\widehat{CBH}=\widehat{CQA}$

hay AQCD là tứ giác nội tiếp 
Tâm (ACD) thuộc trung trực AQ cố định




#662540 Chứng minh $MK=MA$

Posted by Kamii0909 on 20-11-2016 - 19:52 in Hình học

Ta có $MK^2=MO^2-R^2=MH^2+HO^2$
$MA^2=MH^2+HA^2$
Gọi $OA$ cắt $BC$ tại $Q$. Khi đó $HA=HQ$
Khi đó ta phải chứng minh
$HO^2-HA^2=R^2$
$\Leftrightarrow (HO-HQ)(HO+HA)=R^2$
$\Leftrightarrow OQ.OA=OB^2$
Đẳng thức cuối là cơ bản.



#661252 Tìm số nguyên tố p thỏa mãn $p= 2x^{2}-1;p^{2}= 2y^...

Posted by Kamii0909 on 09-11-2016 - 15:02 in Số học

Dễ dàng chứng minh được $ x \leq y \leq p$ và $2 \leq p$
Trừ từng vế 2 phương trình
$p(p-1)=2(y-x)(x+y)$
Suy ra $p|2(y-x)(y+x)$
Mà $2 < $ và $y-x < p $ nên $p|x+y$.
Lại có $x+y < 2p$ nên $x+y=p$
Thay ngược lên có $p-1=2y-2x$
Tới đây dễ rồi. Đơn thuần là giải hệ thôi.
Có $y=3x-1$ và $x^2+2xy=y^2-1$
Thay vào ra $p=7,x=2,y=5$



#668867 Cho $\bigtriangleup ABC$ nội tiếp đường tròn tâm $O....

Posted by Kamii0909 on 19-01-2017 - 17:31 in Hình học

Nếu từ điểm M mà hạ MH vuông góc với BC thì đường AH là đường trong bài P17 của thày Hùng ở tạp chí Pi số 1!

Ùi  :D May quá  :closedeyes: Tý thì e lỡ post lời giải bài P.17  :icon6:  :icon6:  :icon6:  




#670560 Chứng minh rằng: $\left | \frac{a^{3}-b^{...

Posted by Kamii0909 on 06-02-2017 - 19:24 in Bất đẳng thức và cực trị

Cho a, b, c là các số thực dương. Chứng minh rằng:
$\left | \frac{a^{3}-b^{3}}{a+b}+\frac{b^{3}-c^{3}}{b+c}+\frac{c^{3}-a^{3}}{c+a} \right |\leqslant \frac{1}{4}\left [(a-b)^{2}+(b-c)^{2}+(c-a)^{2} \right ]$

Hằng số tốt nhất cho bất đẳng thức này khá xấu và có thể tìm bằng dồn biến toàn miền.
Cho $$a=0,b=2,c=1+ \sqrt{3}+\sqrt{2} \cdot 3^\frac{1}{4}$$ thì $k \geq \sqrt{\dfrac{2 \cdot \sqrt{3}-9}{9}}$



#670339 Chứng minh rằng: $\left | \frac{a^{3}-b^{...

Posted by Kamii0909 on 29-01-2017 - 14:07 in Bất đẳng thức và cực trị

Có vẻ như $\frac{1}{4}$ chưa phải hằng số tốt nhất.
Bình phương lên, điều phải chứng minh tương đương.
$$4\prod(a-b)^2 (ab+bc+ca)^2 \leq \prod (a+b)^2 (\sum a^2-bc)^2$$
Đổi về pqr.
$$ \dfrac{4q^2}{27} [4(p^2-3q)^3 -(2p^3-9pq+27r)^2] \leq (pq-r)^2(p^2-3q)^2 $$
$$L.H.S \leq \dfrac{16q^2(p^2-3q)^3}{27}$$
Ta quy điều phải chứng minh về
$$\dfrac{16q^2(p^2-3q)}{27} \leq (pq-r)^2$$
Có $$pq-r \geq \dfrac{8pq}{9}$$
Thay vào và biến đổi, bất đẳng thức tương đương với
$$q^2(\frac{p^2}{3} +3q) \geq 0$$
Hiển nhiên đúng.



#663699 Chứng minh CD vuông góc OE

Posted by Kamii0909 on 03-12-2016 - 14:48 in Hình học

Còn 1 cách nữa suy nghĩ thêm đi :D

Cách khác.
Ta nhắc lại không chứng minh bổ đề quen thuộc sau.
Cho tứ giác nội tiếp $ABCD$ có 2 tiếp tuyến tại $B,D$ và $AC$ đồng quy. Khi đó 2 tiếp tuyến tại $A,C$ và $BD$ cũng đồng quy.(Tứ giác điều hòa)
Trở lại bài toán. Gọi $P$ là giao điểm $DC$ và $(O)$.
Khi đó theo bổ đề $EP$ là tiếp tuyến của $(O)$.
Từ đó dễ dàng dẫn đến đpcm.



#663696 Chứng minh CD vuông góc OE

Posted by Kamii0909 on 03-12-2016 - 14:18 in Hình học

Bài toán vẫn đúng trong trường hợp $A$ không là trung điểm $OD$.
Dễ dàng chứng minh được $\Delta OBD \sim \Delta ECB$
Nên $\frac{BO}{CE}=\frac{BD}{BC}=\frac{CO}{CE}$
Kết hợp với $\widehat{ECO}=\widehat{DBO}$ ta thu được $\Delta COE \sim \Delta BDC$ từ đó dễ dàng có đpcm.



#674391 $100+9abc \geq 17(ab+bc+ca)$

Posted by Kamii0909 on 15-03-2017 - 23:10 in Bất đẳng thức và cực trị

Cho $a,b,c \in [1,2]$ thoả $a+b+c=5$. 

Chứng minh rằng 

$$100+9abc \geq 17(ab+bc+ca)$$




#669788 $ \sum \sqrt{a+b+\sqrt{ca}+\sqrt{cb}} \geq k(...

Posted by Kamii0909 on 24-01-2017 - 22:29 in Bất đẳng thức - Cực trị

Cho các số thực không âm $a,b,c$ thỏa mãn $a^2+b^2+c^2=2(ab+bc+ca)$
Tìm hằng số k tốt nhất sau cho bất đẳng thức sau luôn đúng $$ \sum \sqrt{a+b+\sqrt{ca}+\sqrt{cb}} \geq k(\sum \sqrt{a})$$



#658284 Tìm GTNN của biểu thức: $a^3+b^3+c^3$

Posted by Kamii0909 on 18-10-2016 - 14:13 in Bất đẳng thức và cực trị

Câu 1: Cho 3 số $a, b,c$ không âm và $a+b+c=3$

Tìm GTNN của biểu thức: $a^3+b^3+c^3$

Câu 2: Cho 3 số $a, b,c$ không âm và $a+b+c=3$

Tìm GTNN của biểu thức:$\sqrt[3]{ab}+\sqrt[3]{bc}+\sqrt[3]{ca}$

Câu 2 

Min=0 khi a=b=0,c=3 và các hoán vị
Nếu a,b,c không lớn hơn 2 thì min=$\sqrt[3]{2}$
Max=3
Theo bđt Holder

$\left ( \sum a \right )\left ( \sum b \right )(1+1+1)\geq \left ( \sum \sqrt[3]{ab} \right )^{3}\Rightarrow \sum \sqrt[3]{ab}\leq 3$

Bài ảo qúa  :wacko:  :wacko:  Bạn check lại đề được không ?? @@




#662811 Có số nguyên dương n nào thỏa mãn ${3^n} + 2003\,\,...

Posted by Kamii0909 on 23-11-2016 - 14:42 in Số học

Ta sẽ chứng minh không tồn tại.
Thật vậy ta phải có
$8|3^n+3$
Với $n=2k,8|3^n-1$
Với $n=2k+1,8|3^n-3$
Từ đó có đpcm.



#660683 25≤ MN2 + NP2 + PQ2 + QM2 ≤ 50

Posted by Kamii0909 on 05-11-2016 - 15:51 in Hình học

Sai rồi bạn. Đâu có cơ sở gì cho bạn xét các TH đặc biệt đâu. M,N,P,Q chạy thoải mái mà.
Mình nghĩ là làm thế này.
Theo định lý Pytago
$MN^2+NP^2+PQ^2+QM^2=(AM^2+MB^2)+(BN^2+NC^2)+(CP^2+PD^2)+(QD^2+QA^2)$
Theo bất đẳng thức Cauchy-Schwarz
$AM^2+MB^2 \geq \frac{1}{2} (AM+MB)^2 = \frac{1}{2} AB^2$
Cộng các bđt tương tự có min =25.
Ta có $AM^2+BM^2 \leq (AM+MB)^2=AB^2$
Cộng lại max =50
Min xảy ra khi M,N,P,Q là các trung điểm
Max xảy ra khi M,N,P,Q trùng A,B,C,D



#661753 $QD$ chia đôi $IH$

Posted by Kamii0909 on 13-11-2016 - 11:49 in Hình học

Đây là bài thi IMO 2010. Có khác nhiều cách làm.




#672103 $ \sum \dfrac{a^2}{b+c}+6(ab+bc+ca) \geq \dfrac{5}{2...

Posted by Kamii0909 on 19-02-2017 - 17:39 in Bất đẳng thức - Cực trị

Chứng minh bất đẳng thức sau với $a,b,c \geq 0,a+b+c=1, k=\dfrac{8}{27} ( 5 \sqrt{10}-13)$
$$ \sum \dfrac{a^2}{b+c}+6(ab+bc+ca) \geq \dfrac{5}{2} +k \dfrac{\sum (a^2b-ab^2)^2}{(a^2+b^2+c^2)^2}$$



#689905 giải đáp phương trình hàm

Posted by Kamii0909 on 08-08-2017 - 14:42 in Phương trình hàm

Sai nhé. 
Không hiểu bạn tìm kiểu gì từ $g(x+1)=(2-a)g(x)+a$ mà ra được $g(x) =$ cái hàm kì dị ấy.
Mà tuyệt đối thử lại cũng không TM luôn.
Hơn nữa $g(0)=0,g(1)=1$ và $g(x+1)=g(x)+1$ chỉ kết luận được $g(x)=x, \forall x \in \mathbb{Z}$

Lời giải bài này như sau: 
$P(x,y) : f(x+y)+f(x)f(y)=f(xy)+f(x)+f(y)$
$P(x,0) : f(x)+f(x)f(0)=f(0)+f(x)+f(0) \Leftrightarrow f(x)f(0) = 2f(0)$
Nếu $f(0) \neq 0$ thì $\boxed{f(x)=2,\forall{x \in \mathbb{R}}}$
Xét $f(0)=0$
$P(x,1) : f(x+1)+f(x)f(1)=f(x)+f(x)+f(1) \Leftrightarrow f(x+1)=f(x) \left[ 2-f(1) \right] +f(1)$
$P(x+1,1) : f(x+2)=f(x) \left[ 2-f(1) \right]^2 +3f(1)-f(1)^2$

$P(1,1) : f(2)=3f(1)-f(1)^2$
$P(x,2) : f(x+2)+f(x)f(2)=f(2x)+f(x)+f(2) \Leftrightarrow f(x) \left[ 2-f(1) \right]^2 +3f(1)-f(1)^2+f(x)f(2)=f(2x)+f(x)+f(2) \Leftrightarrow f(2x)= \left[ 3-f(1) \right]f(x)=af(x)$
$P(2x,2) : f(4x)=a^2f(x)$

$P(2x,2y) -a P(x,y) : (a^2-a)f(x)f(y)=(a^2-a)f(xy)$

Nếu $a=1 \Leftrightarrow P(x,1) : f(x+1)=0 \Leftrightarrow \boxed{f(x)=0,\forall{x \in \mathbb{R}}}$

Nếu $a=0 \Leftrightarrow P(x,2) : f(2x)=0 \Leftrightarrow \boxed{f(x)=0, \forall{x \in \mathbb{R}}}$

Nếu $a^2-a \neq 0$ thì ta có hệ 

$\left\{\begin{matrix} f(x)f(y)=f(xy)\\ f(x)+f(y)=f(x+y) \end{matrix}\right.$
Hệ PTH này quen thuộc và có nghiệm là $ \boxed{ f(x)=0,\forall{x \in \mathbb{R}}}$ hoặc $ \boxed{ f(x)=x,\forall{x \in \mathbb{R}}}$




#664004 Xác định vị trí của điểm $M$ để biểu thức: $P=\frac{...

Posted by Kamii0909 on 06-12-2016 - 20:35 in Hình học

Bài 1.
Xét TH $M$ thuộc cung nhỏ $AD$. Các TH còn lại chứng minh tương tự.
Lấy $G$ trên $AC$ sao cho $\widehat{BMC}=\widehat{AMG}$
Dễ có $\Delta BMC \sim \Delta AMG$ và $\Delta AMB \sim GMC$
Từ đó $\frac{AC}{ME}=\frac{AG}{ME} +\frac{GC}{ME}= \frac{BC}{MD}+ \frac{AB}{MF}$
Từ đó $P=\frac{2AC}{ME}$.
Dễ thấy $P$ không tồn tại GTNN.
Ở đây GTLN $P$ khi $MA=MC$.